Math, asked by Suryanshg, 11 months ago

(p+q)3-(p-q)3-6q(p2-q2)

Answers

Answered by Studentshelper
4
p3+q3-p3+q3-6qp2-6q3=2q3-6q3 -6qp2
= -4q3-6qp2
I hope I have done right
If this was helpful please mark it brainliest

Suryanshg: it's not match with ans
Studentshelper: What is the right answer then
Suryanshg: -6p3+2q3+6p2q+6pq2
Answered by Manyagupta7275
5

I think it will be done like this.
I hope this will help you.
Attachments:

Suryanshg: it doesn't match
Similar questions